Last visit was: 25 Apr 2024, 16:41 It is currently 25 Apr 2024, 16:41

Close
GMAT Club Daily Prep
Thank you for using the timer - this advanced tool can estimate your performance and suggest more practice questions. We have subscribed you to Daily Prep Questions via email.

Customized
for You

we will pick new questions that match your level based on your Timer History

Track
Your Progress

every week, we’ll send you an estimated GMAT score based on your performance

Practice
Pays

we will pick new questions that match your level based on your Timer History
Not interested in getting valuable practice questions and articles delivered to your email? No problem, unsubscribe here.
Close
Request Expert Reply
Confirm Cancel
Manhattan GMAT Online Marketing Associate
Joined: 14 Nov 2013
Posts: 272
Own Kudos [?]: 51 [2]
Given Kudos: 0
Send PM
Manhattan GMAT Online Marketing Associate
Joined: 14 Nov 2013
Posts: 272
Own Kudos [?]: 51 [0]
Given Kudos: 0
Send PM
Manhattan GMAT Online Marketing Associate
Joined: 14 Nov 2013
Posts: 272
Own Kudos [?]: 51 [0]
Given Kudos: 0
Send PM
Manhattan GMAT Online Marketing Associate
Joined: 14 Nov 2013
Posts: 272
Own Kudos [?]: 51 [0]
Given Kudos: 0
Send PM
Re: Manhattan GMAT Blog [#permalink]
Expert Reply
FROM Manhattan GMAT Blog: Manhattan Prep’s Oh What Fun! Celebration Wraps Up

Today wraps up our “Oh What Fun!” celebration! Manhattan Prep’s first-ever 12-day holiday celebration was a great success and we hope that everyone got the chance to enjoy all of our exclusive promos, contests, and free study materials.

If you’ve missed any of this year’s celebration, not to worry, we still have a few promotions going on. Our Black Friday Sale for $150 off all December GMAT classes ends December 15, 2013, with code “BLACKFRIDAY150.” There are still some spots open in our two December online classes, which you can sign up for hereand here. Located in NYC? Come join us in-person for our classes beginning on Dec. 14th and Dec. 15th.

We’re also still offering a free download to our 1-year access GRE Challenge Problem Archive. Remember to enter the code “OWFGRECHALLENGE“ at checkout.

To keep the giving going and the holiday spirit alive, we are very excited to share today’s final celebration! We will be collecting non-perishable food to be donated to New York’s City Harvest now through Dec. 20th. Our goal is to collect a minimum of 200 food items including but not limited to: canned goods, peanut butter, mac-n-cheese, cereal, soups, pastas, etc. We will also be collecting children’s toys to be donated to the Marine Toys for Tots Foundation. Our goal is to collect a minimum of 50 new/unwrapped toysto be distributed to needy children in NYC. Finally, we are also collecting warm clothing to be donated to New York Cares Coat Drive. Our goal is to collect a minimum of 50 new or gently used coats, sweaters and blankets to be distributed to New Yorkers in need. Donations may be made at 138 West 25th St, 7th Fl. New York, NY 10001. Let’s make this season brighter for our community and those in need.

Lastly, be sure to check our Facebook page tomorrow, where we will officially announce this year’s “Oh What Fun!” winners of the free GMAT course and the $250 Amazon Gift Card.

A big thank you to everyone in our social community who participated in this year’s “Oh What Fun!” festivities! Enjoy the remainder of the holiday season and best of luck with your studies!
This Blog post was imported into the forum automatically. We hope you found it helpful. Please use the Kudos button if you did, or please PM/DM me if you found it disruptive and I will take care of it. -BB
Manhattan GMAT Online Marketing Associate
Joined: 14 Nov 2013
Posts: 272
Own Kudos [?]: 51 [0]
Given Kudos: 0
Send PM
Re: Manhattan GMAT Blog [#permalink]
Expert Reply
FROM Manhattan GMAT Blog: Free GMAT Events This Week: December 15- December 21
Here are the free GMAT events we’re holding this week. All times are local unless otherwise specified.

12/15/13- Online - Essay Writing Workshop presented by mbaMission-  8:00AM- 9:30PM (EST)

12/15/13- Online- Free Trial Class- 7:00AM- 10:00AM (EST)

12/16/13-New York, NY-MBA Admissions Myths Destroyed presented by mbaMission- 7:30PM- 9:00PM

12/19/13-  Online - MBA Admissions Myths Destroyed presented by mbaMission- 12:00PM- 1:30PM (EST)

Looking for more free events? Check out our Free Events Listings Page.
This Blog post was imported into the forum automatically. We hope you found it helpful. Please use the Kudos button if you did, or please PM/DM me if you found it disruptive and I will take care of it. -BB
Manhattan GMAT Online Marketing Associate
Joined: 14 Nov 2013
Posts: 272
Own Kudos [?]: 51 [0]
Given Kudos: 0
Send PM
Re: Manhattan GMAT Blog [#permalink]
Expert Reply
FROM Manhattan GMAT Blog: mbaMission B-School Chart of the Week: November 2013 Social Currency Ranking
We’ve invited mbaMission: MBA Admissions Consulting to share their Business School Charts of the Week. Here is their Chart for November 2013 Social Currency Ranking.

Rankings come in all shapes and sizes, but can any ranking truly capture social cachet? For a different perspective on the value of an MBA, we turn to the New York Times society pages, where the editors select and profile promising couples. Each month, we dedicate one B-School Chart of the Week to tallying how alumni from top-ranked business schools are advancing their social currency ranking.

Colder weather and holiday travel seem to have brought about a lull in the New York Times wedding announcements for November. Still, of the 124 total announcements last month, 19 included a business school mention.

Several weddings featured MBA students specifically. For example, Nicholas Tangney, who is a managing director for Lorentzen & Stemoco and is studying for his MBA at New York University’s Stern School of Business, was married to Samantha Lee, a vice president and account manager for consumer products clients at DeVries Global. Similarly,Tracy Massel, a student at Harvard Business School, married Steven Melzer, the director of finance and operations strategy at Expeditionary Learning. Morgan Fauth, a first year at Georgetown’s McDonough School of Business who married research analyst Kevin Patrick Coleman, Jr., was also featured among the wedding announcements.

Notably, Dartmouth Tuck, which has as yet kept a rather low profile in the society pages, gained two mentions last month, raising its year-to-date total from five to seven with the marriage of alumnus and strategy consultant Andrew Brown to Caroline Hockmeyer, and the marriage of alumna and merchandise planning manager Anne Snodgrass to Wharton MBA Zachary Dennett.

Another notable November announcement, though not technically an MBA wedding, was that of Ira Millstein, an adjunct professor and co-chairman of the Millstein Center for Global Markets and Corporate Ownership at Columbia Business School, to former stock trader Susan Frame. Millstein is also a senior partner at Weil, Gotshal & Manges and the former chairman of both the Central Park Conservancy and the American Red Cross of Greater New York.

With a total of approximately 1,284 wedding announcements since January, our year-to-date total of MBA mentions hits 260 this month.

This Blog post was imported into the forum automatically. We hope you found it helpful. Please use the Kudos button if you did, or please PM/DM me if you found it disruptive and I will take care of it. -BB
Manhattan GMAT Online Marketing Associate
Joined: 14 Nov 2013
Posts: 272
Own Kudos [?]: 51 [0]
Given Kudos: 0
Send PM
Re: Manhattan GMAT Blog [#permalink]
Expert Reply
FROM Manhattan GMAT Blog: GMAT Challenge Problem Showdown: December 16, 2013


We invite you to test your GMAT knowledge for a chance to win! Each week, we will post a new Challenge Problem for you to attempt. If you submit the correct answer, you will be entered into that week’s drawing for a free Manhattan GMAT Prep item. Tell your friends to get out their scrap paper and start solving!

Here is this week’s problem:



A set of n identical triangles with angle x° and two sides of length 1 is assembled to make a parallelogram (if n is even) or a trapezoid (if n is odd), as shown. Is the perimeter of the parallelogram or trapezoid less than 10?


To see the answer choices, and to submit your answer, visit our Challenge Problem Showdown page on our site.

Discuss this week’s problem with like-minded GMAT takers on our Facebook page.

The weekly winner, drawn from among all the correct submissions, will receive One Year of Access to our Challenge Problem Archive, AND the OG Archer, AND Our Six Computer Adaptive Tests ($92 value).
This Blog post was imported into the forum automatically. We hope you found it helpful. Please use the Kudos button if you did, or please PM/DM me if you found it disruptive and I will take care of it. -BB
Manhattan GMAT Online Marketing Associate
Joined: 14 Nov 2013
Posts: 272
Own Kudos [?]: 51 [0]
Given Kudos: 0
Send PM
Re: Manhattan GMAT Blog [#permalink]
Expert Reply
FROM Manhattan GMAT Blog: Andrew Yang: “Smart People Should Build Things” Excerpt 2
Below is an excerpt from Andrew Yang‘s new book, Smart People Should Build Things: How to Restore Our Culture of Achievement, Build a Path for Entrepreneurs, and Create New Jobs in America, which comes out in February 2014. Andrew was named Managing Director of Manhattan GMAT in 2006, Chief Executive Officer in 2007, and President in 2010. He left Manhattan GMAT in 2010 to start Venture for America, where he now serves as Founder and CEO.


The Prestige Pathways.  

Let’s imagine a very large company. It is a leader in its industry and much admired by its peers. It invests a tremendous amount of money—literally billions of dollars a year—in identifying, screening, and training its many employees. Those employees who are considered to have high potential are sent to special training programs at substantial additional cost. Happily, these top training programs are considered to be among the best in the world. After these employees complete their training, the company encourages them to choose for themselves the division in which they’d like to work. Employee preferences are deemed to be the most efficient way of deciding who works where.

This seems like a good system, and it works well for a long time. However, perhaps predictably, many of its most highly rated employees eventually become drawn to the finance and legal divisions because these divisions have very effective recruitment arms, are more visible, pay better, and are thought of as providing a more intellectual level of work.  Over time, proportionally fewer of the top recruits go toward the management of the company or the company’s operations.  The company’s basic training division is considered a backwater, with low pay and low recognition. And only a relative handful of employees go toward research and development or the launching of any new products.

Take a second to think about the company described above. What do you think will happen to this company as time passes? And if you think that it’s not set on a path to success, what would you do to fix it? This company reflects, in essence, the economy of the United States of America.

If you are a smart college student and you want to become a lawyer and go to law school, what you must do has been well established. You must go to a good school, get good grades (already accomplished, for many), and take the LSAT (a four-hour skill test). There is no anxiety in divining the requirements, as they are clearly spelled out. Most undergrads, even those with little interest in law school, know what it takes to get in. The path location costs are low.

The same is true if you want to become a doctor. Becoming a doctor is hard, right? Sort of. It is arduous and time-consuming, but it is not hard if you have certain academic abilities. You must take a battery of college courses (organic chemistry being the most infamous and rigorous of them) and do well, study for the MCAT (an eight-hour exam), and spend a summer or even a year caddying for a researcher, doctor, or hospital. These are time-consuming hoop-jumping tasks, to be sure, but anyone with a very high level of academic aptitude can complete them.

If you attend an Ivy League university or similar national institution, legions of suit-wearing representatives from the big-name investment banks and consulting firms will show up at your campus and conduct first-round interviews to fill their ranks each year, even in a down period (as with the recent years following the financial crisis). They will spend millions of dollars enlisting interns and educating the market annually. Most freshmen have no idea what management consulting is, yet seniors can rattle off the distinctions of different firms with little difficulty. All undergraduates have friends in the classes above them who have gone through this process and gained analyst or associate positions at major investment banks and consulting firms.

Again, the requirements are clear: you have to have good grades, be able to perform some cognitive tasks with words and numbers in the form of case studies that you should prepare for and practice, and hopefully look good in a suit. It is also very helpful if you spend a summer in college doing something that can be presented as relating to your professional interest; in many cases it’s necessary that you intern at the employer the summer before your senior year in order to get an offer. Summer internships have become vital for getting jobs in the most selective firms, so the process begins quite early—junior year at the latest. This path requires some early choices, but you don’t have to spend time taking another standardized test. Of course, many of the people who go into finance and consulting take the GMAT and go on to business school.

These structured paths are clearly laid out, and are pursued collectively by many—or most—of the students who have been screened and sorted as the academic and cognitive elite. These “prestige pathways” have become the default options. In 2011, 29 percent of employed Harvard graduates went into finance or consulting, while 19 percent of the class applied to law school and 18 percent applied to medical school.  That’s a majority of the class. California (San Francisco), New York (New York City), and Massachusetts (Boston) were the only states that received over one hundred Harvard grads in 2012, with Illinois (Chicago) and Washington DC, being the only other destinations to receive fifty or more.  The statistics from Yale, Dartmouth, Penn, and other top schools are similar.

Perhaps this is somewhat surprising—wouldn’t college students at these top schools be positioned to blaze their own trails and pursue less conventional routes with the access that they have been given?

Unfortunately, hardworking, academically gifted young people are kind of lazy when it comes to determining direction. If you give them a hoop to jump through, jumping through that hoop can take two, twenty, or two hundred hours, and it won’t make a big difference. But they are quite lazy when it comes to figuring out what path to take or—more profoundly—building their own path. They’re trained to get the grade or ace the application. That is what has made them successful in most every conventional respect each step of the way up to their senior year in college, at the point that this process is well underway.

“It’s doing a process that you’ve done a billion times before,” explains Dylan Matthews, a 2012 Harvard graduate who wrote for the campus newspaper, the Harvard Crimson, before becoming a journalist. He adds, “Everyone who goes to Harvard went hard on the college application process. Applying to Wall Street is much closer to that than applying anywhere else is. There are a handful of firms you really care about, they all have formal application processes that they walk you through, there’s a season when it all happens, all of them come to you and interview you where you live. Harvard students are really good at formal processes like that, and they’re less good at going on Monster or Craigslist and sorting through thousands of job listings from thousands of companies whose reputations they don’t know. Wall Street and consulting (and Teach for America, too) turn applying to jobs into applying to college [again], more or less.

Of course, the same procedural comfort level applies to law school and other graduate programs, and the same mindset pervades competitive campuses around the country.

 

From SMART PEOPLE SHOULD BUILD THINGS by Andrew Yang© 2014 Andrew Yang. Reprinted courtesy of Harper Business, an imprint of HarperCollins Publishers.
This Blog post was imported into the forum automatically. We hope you found it helpful. Please use the Kudos button if you did, or please PM/DM me if you found it disruptive and I will take care of it. -BB
Manhattan GMAT Online Marketing Associate
Joined: 14 Nov 2013
Posts: 272
Own Kudos [?]: 51 [0]
Given Kudos: 0
Send PM
Re: Manhattan GMAT Blog [#permalink]
Expert Reply
FROM Manhattan GMAT Blog: Tackling Multi-Shape Geometry on the GMAT
What do you do when you realize a geometry problem has just popped up on the screen? Try this GMATPrep© problem from the free practice test and then we’ll talk about what to do!



In the figure above, the radius of the circle with center O is 1 and BC = 1. What is the area of triangular region ABC?



What’s your first step? Let’s use this problem as an opportunity to practice the Quant Process.

 



At a glance, you can see that the problem provides a diagram. Draw! Make it big enough that you can add labels as you calculate new pieces of information (and, of course, jot down any information given in the problem).

Finally, write down any formulas you’ll need, as well as whatever the problem asks you to find. Your scrap paper might look something like this:



 

Before you dive in and try to find this height, though, Reflect! Ask yourself whether there are other possible ways to move forward. Sometimes, the “obvious” way turns out not to be the easiest way to proceed.

In particular, this is a “multi-shape” problem: you were given both a triangle and a circle. Why did they include the circle? Pay particular attention to where the two shapes overlap.

Hmm. The hypotenuse of the triangle is also a diameter of the circle. How can you use that to solve?

It turns out that when a triangle is inscribed in a circle (the 3 vertices of the triangle all sit on the circle), and the hypotenuse of that triangle is also a diameter of the circle, then the triangle in question is a right triangle. (This is one of the rules we’re supposed to memorize for the test.)

In this case, the right angle is labeled B. Is that information useful at all? Well, if you’re trying to find the area of a right triangle, then you just need to know the lengths of the two legs: AB and BC. The problem says that BC = 1, so the only unknown is AB.

Now you have a choice: do you think it’ll be easier to find the length of AB or to find the length of the vertical line that you drew in below point B?

Because ABC is a right triangle, it’s easier to find AB. The short leg is 1 and the hypotenuse is 2. Do those numbers match any of the “smart” triangles that you’ve studied? (If not, use the Pythagorean Theorem.)

Yes! These match the 30-60-90 triangle parameters.



The length of AB is 
. Plug this into the area formula:



 

 

 

 

The correct answer is (B). 

Key Takeaways for Multi-Shape Geometry.

(1) Examine the “overlap” between the shapes. Most likely, some rule about that connection exists and this rule will help make the problem easier to solve.

(2) Draw! This is key for any geometry problem, but especially so for multi-shape problems. There are too many moving parts; you need to keep track of everything in a clear way.

(3) Remember our Quant Process: Reflect before you Work! In this case, the first, more obvious path would have been a lot more difficult to execute. Reflecting for a moment allowed you to notice the connection between the circle and the triangle. The subsequent solution path turned out to be much more straightforward.



* GMATPrep© questions courtesy of the Graduate Management Admissions Council. Usage of this question does not imply endorsement by GMAC

 

 
This Blog post was imported into the forum automatically. We hope you found it helpful. Please use the Kudos button if you did, or please PM/DM me if you found it disruptive and I will take care of it. -BB
Manhattan GMAT Online Marketing Associate
Joined: 14 Nov 2013
Posts: 272
Own Kudos [?]: 51 [0]
Given Kudos: 0
Send PM
Re: Manhattan GMAT Blog [#permalink]
Expert Reply
FROM Manhattan GMAT Blog: GMAT Challenge Problem Showdown: December 23, 2013


We invite you to test your GMAT knowledge for a chance to win! Each week, we will post a new Challenge Problem for you to attempt. If you submit the correct answer, you will be entered into that week’s drawing for a free Manhattan GMAT Prep item. Tell your friends to get out their scrap paper and start solving!

Here is this week’s problem:

A pharmacy must purchase a set of n metal weights, each weighing an integer number of grams, such that all integer weights from 1 to 300 grams (inclusive) can be made with a combination of one or more of the weights. What is the minimum number of metal weights that the pharmacy must purchase?


To see the answer choices, and to submit your answer, visit our Challenge Problem Showdown page on our site.

Discuss this week’s problem with like-minded GMAT takers on our Facebook page.

The weekly winner, drawn from among all the correct submissions, will receive One Year of Access to our Challenge Problem Archive, AND the OG Archer, AND Our Six Computer Adaptive Tests ($92 value).
This Blog post was imported into the forum automatically. We hope you found it helpful. Please use the Kudos button if you did, or please PM/DM me if you found it disruptive and I will take care of it. -BB
Manhattan GMAT Online Marketing Associate
Joined: 14 Nov 2013
Posts: 272
Own Kudos [?]: 51 [0]
Given Kudos: 0
Send PM
Re: Manhattan GMAT Blog [#permalink]
Expert Reply
FROM Manhattan GMAT Blog: The 4 Math Strategies Everyone Must Master, part 2

Last time, we talked about the first 2 of 4 quant strategies that everyone must master: Test Cases and Choose Smart Numbers.

Today, we’re going to cover the 3rd and 4th strategies. First up, we have Work Backwards. Let’s try a problem first: open up your Official Guide, 13th edition (OG13), and try problem solving #15 on page 192. (Give yourself about 2 minutes.)

I found this one by popping open my copy of OG13 and looking for a certain characteristic that meant I knew I could use the Work Backwards technique. Can you figure out how I knew, with just a quick glance, that this problem qualified for the Work Backwards strategy? (I’ll tell you at the end of the solution.)

For copyright reasons, I can’t reproduce the entire problem, but here’s a summary: John spends 1/2 his money on fruits and vegetables, 1/3 on meat, and 1/10 on treats from the bakery. He also spends $6 on candy. By the time he’s done, he’s spent all his money. The problem asks how much money he started out with in the first place.

Here are the answer choices:

“(A) $60

“(B) $80

“(C) $90

“(D) $120

“(E) $180”

Work Backwards literally means to start with the answers and do all of the math in the reverse order described in the problem. You’re essentially plugging the answers into the problem to see which one works. This strategy is very closely tied to the first two we discussed last time—except, in this instance, you’re not picking your own numbers. Instead, you’re using the numbers given in the answers.

In general, when using this technique, start with answer (B) or (D), your choice. If one looks like an easier number, start there. If (C) looks a lot easier than (B) or (D), start with (C) instead.

This time, the numbers are all equally “hard,” so start with answer (B). Here’s what you’re going to do:

(B) $80

 

F + V (1/2)

M (1/3)

B (1/10)

C $6

Add?

(B) $80

$40

…?

$6

Set up a table to calculate each piece. If John starts with $80, then he spends $40 on fruits and vegetables. He spends… wait a second! $80 doesn’t go into 1/3 in a way that would give a dollar-and-cents amount. It would be $26.66666 repeating forever. This can’t be the right answer!

Interesting. Cross off answer (B), and glance at the other answers. They’re all divisible by 3, so we can’t cross off any others for this same reason.

Try answer (D) next.

 

F + V (1/2)

M (1/3)

B (1/10)

C $6

Add to?

(B) $80

$40

…?

$6

?

(D) $120

$60

$40

$12

$6

$118

 

In order for (D) to be the correct answer, the individual calculations would have to add back up to $120, but they don’t. They add up to $118.

Okay, so (D) isn’t the correct answer either. Now what? Think about what you know so far. Answer (D) didn’t work, but the calculations also fell short—$118 wasn’t large enough to reach the starting point. As a result, try a smaller starting point next.

 

F + V (1/2)

M (1/3)

B (1/10)

C $6

Add?

(B) $80

$40

…?

$6

?

(D) $120

$60

$40

$12

$6

$118

(C) $90

$45

$30

$9

$6

$90

 

It’s a match! The correct answer is (C).

Now, why would you want to do the problem this way, instead of the “straightforward,” normal math way? The textbook math solution on this one involves finding common denominators for three fractions—somewhat annoying but not horribly so. If you dislike manipulating fractions, or know that you’re more likely to make mistakes with that kind of math, then you may prefer to work backwards.

Note, though, that the above problem is a lower-numbered problem. On harder problems, this Work Backwards technique can become far easier than the textbook math. Try PS #203 in OG13. I would far rather Work Backwards on this problem than do the textbook math!

So, have you figured out how to tell, at a glance, that a problem might qualify for this strategy?

It has to do with the form of the answer choices. First, they need to be numeric. Second, the numbers should be what we consider “easy” numbers. These could be integers similar to the ones we saw in the above two problems. They could also be smaller “easy” fractions, such as 1/2, 1/3, 3/2, and so on.

Further, the question should ask about a single variable or unknown. If it asks for x, or for the amount of money that John had to start, then Work Backwards may be a great solution technique. If, on the other hand, the problem asks for x – y, or some other combination of unknowns, then the technique may not work as well.

(Drumroll, please) We’re now up to our fourth, and final, Quant Strategy that Everyone Must Master. Any guesses as to what it is? Try this GMATPrep© problem.



“In the figure above, the radius of the circle with center O is 1 and BC = 1. What is the area of triangular region ABC?



If the radius is 1, then the bottom line (the hypotenuse) of the triangle is 2. If you drop a line from point B to that bottom line, or base, you’ll have a height and can calculate the area of the triangle, since A = (1/2)bh.

You don’t know what that height is, yet, but you do know that it’s smaller than the length of BC. If BC were the height of the triangle, then the area would be A = (1/2)(2)(1) = 1. Because the height is smaller than BC, the area has to be smaller than 1. Eliminate answers (C), (D), and (E).

Now, decide whether you want to go through the effort of figuring out that height, so that you can calculate the precise area, or whether you’re fine with guessing between 2 answer choices. (Remember, unless you’re going for a top score on quant, you only have to answer about 60% of the questions correctly, so a 50/50 guess with about 30 seconds’ worth of work may be your best strategic move at this point on the test!)

The technique we just used to narrow down the answers is one I’m sure you’ve used before: Estimation. Everybody already knows to estimate when the problem asks you for an approximate answer. When else can (and should) you estimate?

Glance at the answers. Notice anything? They can be divided into 3 “categories” of numbers: less than 1, 1, and greater than 1.

Whenever you have a division like this (greater or less than 1, positive or negative, really big vs. really small), then you can estimate to get rid of some answers. In many cases, you can get rid of 3 and sometimes even all 4 wrong answers. Given the annoyingly complicated math that sometimes needs to take place in order to get to the final answer, your best decision just might be to narrow down to 2 answers quickly and then guess.

Want to know how to get to the actual answer for this problem, which is (B)? Take a look at the full solution here.

The 4 Quant Strategies Everyone Must Master

Here’s a summary of our four strategies.

(1) Test Cases.

-      Especially useful on Data Sufficiency with variables / unknowns. Pick numbers that fit the constraints given and test the statement. That will give you a particular answer, either a value (on Value DS) or a yes or no (on Yes/No DS). Then test another case, choosing numbers that differ from the first set in a mathematically appropriate way (e.g., positive vs. negative, odd vs. even, integer vs. fraction). If you get an “always” answer (you keep getting the same value or you get always yes or always no), then the statement is sufficient. If you find a different answer (a different value, or a yes plus a no), then that statement is not sufficient.

-      Also useful on “theory” Problem Solving questions, particularly ones that ask what must be true or could be true. Test the answers using your own real numbers and cross off any answers that don’t work with the given constraints. Keep testing, using different sets of numbers, till you have only one answer left (or you think you’ve spent too much time).

(2) Choose Smart Numbers.

-      Used on Problem Solving questions that don’t require you to find something that must or could be true. In this case, you need to select just one set of numbers to work through the math in the problem, then pick the one answer that works.

-      Look for variable expressions (no equals or inequalities signs) in the answer choices. Will also work with fraction or percent answers.

(3) Work Backwards.

-      Used on Problem Solving questions with numerical answers. Most useful when the answers are “easy”—small integers, easy fractions, and so on—and the problem asks for a single variable. Instead of selecting your own numbers to try in the problem, use the given answer choices.

-      Start with answer (B) or (D). If a choice doesn’t work, cross it off but examine the math to see whether you should try a larger or smaller choice next.

(4) Estimate.

-      You’re likely already doing this whenever the problem actually asks you to find an approximate answer, but look for more opportunities to save yourself time and mental energy. When the answers are numerical and either very far apart or split across a “divide” (e.g., greater or less than 0, greater or less than 1), you can often estimate to get rid of 2 or 3 answers, sometimes even all 4 wrong answers.

 

The biggest takeaway here is very simple: these strategies are just as valid as any textbook math strategies you know, and they also require just as much practice as those textbook strategies. Make these techniques a part of your practice: master how and when to use them, and you will be well on your way to mastering the Quant portion of the GMAT!

 

 
This Blog post was imported into the forum automatically. We hope you found it helpful. Please use the Kudos button if you did, or please PM/DM me if you found it disruptive and I will take care of it. -BB
Manhattan GMAT Online Marketing Associate
Joined: 14 Nov 2013
Posts: 272
Own Kudos [?]: 51 [0]
Given Kudos: 0
Send PM
Re: Manhattan GMAT Blog [#permalink]
Expert Reply
FROM Manhattan GMAT Blog: Andrew Yang: “Smart People Should Build Things” Excerpt 3
Below is an excerpt from Andrew Yang‘s new book, Smart People Should Build Things: How to Restore Our Culture of Achievement, Build a Path for Entrepreneurs, and Create New Jobs in America, which comes out in February 2014. Andrew was named Managing Director of Manhattan GMAT in 2006, Chief Executive Officer in 2007, and President in 2010. He left Manhattan GMAT in 2010 to start Venture for America, where he now serves as Founder and CEO.


The Prestige Pathways Part II.  

You could ask, so what if our talented young people all march off to become lawyers, doctors, bankers, and consultants? Isn’t that what smart people are supposed to do?\

There are a few problems with this stance. First, the degree to which the recruitment infrastructure exists is a relatively recent phenomenon. Bain and Company, a premier management consulting firm, wasn’t founded until 1973—now it employs over 5,000 talented people and recruits hundreds per year. The financial services industry has mushroomed in size, with Wall Street firms employing 191,800 at their peak in 2008, up from only 65,300 in 1975. The growth in professional services has given rise to an accompanying set of recruitment pipelines only in the past several decades.

Yet the allocation of talent is a zero-sum game. If the academically gifted are funneled in higher numbers toward finance and consulting, then lesser numbers are going into other areas, such as the operation of companies, startups, and early-stage enterprises. In the United States, companies with fewer than 500 employees account for almost two-thirds of net new jobs and generate thirteen times more new patents per employee than do large firms. If the US economy had generated as many startups each year for 2009–12 as it had in 2007, the country would have produced almost 2.5 million new jobs by 2013. If we’re interested in spurring long-term job growth, we want as much talent as possible heading to new firms so that more of them can succeed, expand, and hire more people.

Further, the current talent flows have a pronounced regional bias. The hubs for financial services and consulting are New York, San Francisco, Boston, Chicago, Los Angeles, and Washington, DC, and these cities are magnets for the preponderance of top university graduates. Meanwhile, dozens of other US cities and communities are home to promising growth companies that don’t have the talent they need to develop and expand. Companies in Detroit, New Orleans, Las Vegas, Providence, Baltimore, Cleveland, and other cities are poised to hire and to provide new opportunities and products. Yet our national university graduates are being consistently channeled elsewhere.*

Professional services industries like finance, consulting, and legal services are, by definition, meta-industries. That is, they serve to help large companies raise money, buy and sell each other, reorganize, implement new systems, conduct complex transactions, and so forth. They are dependent on companies coming into being and becoming big enough to hire them. The economy needs more companies to start, grow, and thrive in order for the service organizations themselves to prosper. For example, if Mark Zuckerberg had become an investment banker or gone to work in a bank’s information technology department, then the bankers wouldn’t have had Facebook to take public. It’s actually far better for the investment banks (and everyone else) that instead of heading in their direction, he started his own company.

Another issue is that professional paths aren’t always the right fit. Everyone reading this knows a host of former lawyers, bankers, consultants, academics, or doctors for whom the work or environment was not right, many of whom eventually left the profession or stuck around halfheartedly. This represents a massive social cost. Instead of an army of bright college graduates, we are left with an array of often indebted former professionals who are only starting years later what should have been their first act. Some find roles that fit. But for most this transition is not seamless; there are often time-consuming stumbles and periods of exploration before a new path is forged or found—if one is found.

Last, and perhaps most important, professional services socialize individuals in ways that are not conducive to their ability to contribute in other ways. All of us, and particularly young people, have a tendency to view ourselves and our natures as static: you’ll choose to do something for a few years, and you’ll still be the same you. This isn’t the case. Spending your twenties traveling four days a week, interviewing employees, and writing detailed reports on how to cut costs will change you, as will spending years editing contracts and arguing about events that will never come to pass, or years producing Excel spreadsheets and moving deals along. After a while, regardless of your initial motivations, your lifestyle and personality will change to fit your role. You will become a better dispenser of well-presented recommendations, or editor of contracts, or generator of financial projections. And you will in all likelihood become less good at other things. You will not be the same person you were when you started.

It is no accident that many of those we regard as our most productive individuals—Bill Gates, Steve Jobs, Jeff Bezos, Howard Schultz, Jack Dorsey, Reid Hoffman, Larry Page, Sergey Brin, and the like—were not products of our professional paths. Michael Dell actually entered the University of Texas intending to go to medical school.  He probably would have made a fine doctor. But thanks to him over 100,000 people are now working at his namesake company, both in Texas and around the world.

* One could argue that our national university system has become a de facto talent drain for much of the country. Many states and communities send their top students away to great schools, never to hear from them again.

 

From SMART PEOPLE SHOULD BUILD THINGS by Andrew Yang© 2014 Andrew Yang. Reprinted courtesy of Harper Business, an imprint of HarperCollins Publishers.
This Blog post was imported into the forum automatically. We hope you found it helpful. Please use the Kudos button if you did, or please PM/DM me if you found it disruptive and I will take care of it. -BB
Manhattan GMAT Online Marketing Associate
Joined: 14 Nov 2013
Posts: 272
Own Kudos [?]: 51 [0]
Given Kudos: 0
Send PM
Re: Manhattan GMAT Blog [#permalink]
Expert Reply
FROM Manhattan GMAT Blog: Monthly GMAT Challenge Problem Showdown: January 13, 2013


We invite you to test your GMAT knowledge for a chance to win! The second week of every month, we will post a new Challenge Problem for you to attempt. If you submit the correct answer, you will be entered into that month’s drawing for free Manhattan GMAT prep materials. Tell your friends to get out their scrap paper and start solving!

Here is this month’s problem:

If p, q, and r are different positive integers such that p + q + r = 6, what is the value of x ?

(1) The average of xp and xq is xr.

(2) The average of xp and xr is not xq.


To see the answer choices, and to submit your answer, visit our Challenge Problem Showdown page on our site.

Discuss this month’s problem with like-minded GMAT takers on our Facebook page.

The monthly winner, drawn from among all the correct submissions, will receive One Year of Access to our Challenge Problem Archive, AND the OG Archer, AND Our Six Computer Adaptive Tests ($92 value).
This Blog post was imported into the forum automatically. We hope you found it helpful. Please use the Kudos button if you did, or please PM/DM me if you found it disruptive and I will take care of it. -BB
Manhattan GMAT Online Marketing Associate
Joined: 14 Nov 2013
Posts: 272
Own Kudos [?]: 51 [0]
Given Kudos: 0
Send PM
Re: Manhattan GMAT Blog [#permalink]
Expert Reply
FROM Manhattan GMAT Blog: New Year’s Resolution: Get Your Score!

Whether you’ve been studying for a while or are just getting started, let’s use the New Year as an opportunity to establish or renew your commitment to getting your desired GMAT score.

In the first half of this 2-part series, we’ll talk about how to get started—or re-started—on your GMAT prep. In the second half, we’ll talk about how to learn.

Wherever you are in your study, you need a plan, and the first important thing to learn is that no plan is static. No plan exists that says, “Here’s what you’ll do from Day 1 right up until Test Day.” (No good plan, at least!)

Most people can start off in very similar ways, but at some point down the road, you’re going to have to customize based on your own needs. We’ll talk more about that in the second installment of this series.

Start keeping a GMAT Journal. Get a notebook, open up a file on your computer, or start a blog (though I’d recommend making it a private blog, with an audience of just you). Write something in your GMAT Journal every day.* Don’t write everything, but do write:

(1) what you did that day*

(2) the two or three most important things you learned (such as “how do I know when to cut myself off on a quant problem?”)

(3) one or two things you want to review at a later date (such as “review modifier rules in 2 weeks.”)

* Note that, on some days, you’ll write “Relaxed / took my Earned GMAT Break.” Don’t burn yourself out!

1: Set Your Goal
(note: this section is NOT just for new students—keep reading even if you’ve been studying for a while or already know your goal score!)

You need to know your current score and the score level that will make you competitive at the schools to which you plan to apply. These two numbers will give you an idea of how much improvement you will need and may affect your prep plans, including the length of time you plan to spend and whether you work on your own.

If you haven’t already (within the past 4-ish weeks), take a practice CAT in conditions that simulate the actual exam as much as possible. Do the essay and IR sections. The mental effort it takes to do these sections can affect your performance on quant and verbal, so don’t skip them because you don’t care about the IR and essay scores. Take two 8-minute breaks, one after IR and one after the quantitative section. Don’t answer the phone, don’t eat or drink except during the breaks, and so on—basically make it as close to the real test as you can.

Many prep companies offer practice exams, so you have plenty of choices, but you do need to make sure that the exam does several things. First, the quant and verbal sections should be adaptive, just like the real test. Second, the test should record the time you spend on each individual question—timing is a major factor on the GMAT. Third, it should offer score reports that give you tons of data on your strengths and weaknesses.

GMATPrep® exams (from the makers of the real test) are great in general but do not give you the 2nd and 3rd items on this list, so don’t use a GMATPrep CAT for this exercise. Save GMATPrep for closer to the time you plan to take the real test.

Next, go to the websites of the schools to which you want to apply (or may want to apply) and find the GMAT statistics for the most recent admitted students.

Record your practice score and the school statistics in your journal. As a general rule, your GMAT score is a “plus” for you if you are at or above the median for a given school, so ideally your goal score should be at or above the median for your schools.

How far are you from your goal? The further you are or the higher your goal score is, the longer you will likely need to prep for the exam. Most people prep for between 2.5 and 4 months (though obviously the length of time can vary). It’s reasonable, though, to aim for a minimum of 2 months unless you don’t need very much improvement at all.

2: Diagnose Your Strengths and Weaknesses?
Next, use your test results to figure out your strengths and weaknesses in terms of both content and timing. You can use this article to help analyze a ManhattanGMAT CAT. Take notes on paper, then summarize your analysis in your journal. (Note: analysis is not the same thing as data. The data tells you what happened. Your analysis tells you why you think it happened and what you plan to do about it in future. Start by summarizing the data, but don’t forget to take the next step and analyze.)

Also, what is your optimal learning style? Think back to undergrad. Did you do best when you had a small classroom of comrades with whom you shared the adventures of learning? Or did you excel when you worked on your own, or possibly met individually with your professor or TA? At work today, does it energize you to work with a group or do you focus better via one-on-one interactions? Do you prefer to do most of your work on your own or with others?

The answers to those questions will help you determine whether to study on your own, find other students with whom to study, take an organized class, or find a private tutor. There’s no one right way—there’s only the best way for you.

3: Plan Your Schedule
Now that you know your strengths and weaknesses, you can use that info to help determine a rough timeframe. The ideal is to work without an external deadline (e.g., a school application deadline). You set a general timeframe / deadline for yourself and get started, but you’re able to take more time if needed, since you don’t absolutely have to take the test by a certain date.

If you are working against a deadline, though, then you have to plan more carefully. Be aware that you may also have to decide, at some point, to lower your goal score in order to take the test by a certain date.

Most people initially underestimate the amount of time they’ll need to study. Plus, we’re talking about a time period of 2 months or longer; it’s very unlikely that you can pick an exact date (or even an exact week) so far ahead of time. If you have the luxury of time, set yourself a general timeframe, but start to think about specific test dates only when your practice CAT scores start to get into the range you want.

Here’s how to set your overall timeframe.

1. Primary Study Period.

You’ll set a rough amount of time that you’re likely going to need for primary studying (that is, the time you take to master the material, not including a comprehensive final review). Be aware that this rough timeframe is likely to change as you see how fast you make progress.

For most people, primary study will take 8 to 16 weeks, though it may be a bit shorter if you’ve taken the test before and you’re not aiming for a significant (> 50 points) improvement. If, on the other hand, you’re starting from scratch and you want an extra-high improvement (>150 points), or you have a crazy schedule and can’t study very much /often, you may need more than 16 weeks. Also, if you take a class, your primary study will be at least the length of the class plus some additional time.

2. Review Period.

You will also need to set aside time for review after you finish your primary study and before you take the test. Most people spend 2 to 6 weeks on a comprehensive review after they finish their primary study. If you’re going to do this in 2 weeks, you’ll need to be able to spend at minimum 10 hours per week. Pick a rough target based on what you know of your schedule for now but, again, be aware that this could change in future.

You also need to factor in two other things that will affect your study timeframe:

3. Buffer.

You may not get the test score that you want—occasionally, people even get sick right in the testing room. It’s smart to leave time to take the test a second time, if necessary. You are only allowed to take the GMAT once in a 31-day period (and 5 times a year), so plan this “buffer” time into your prep schedule.

You may also want to include a couple of extra weeks of study time as an additional buffer, just in case. Work gets busy, people get sick, we procrastinate… things happen.

4. Hard Deadlines.

You will, of course, have to meet the application deadlines of your selected schools. If you can plan ahead, it’s preferable to get the test out of the way well before you have to start filling out the applications themselves. (Keep in mind that your GMAT score is valid for 5 years, so you can get started very early!)

4: Gather Your Resources
There are tons of resources available to help you get ready for the GMAT. If you take a course or work with some structured program, the materials should already be determined for you. Otherwise, you’ll have to figure out what works best for you.

In general, there are three major categories of necessary resources:

1. Test content and methodology.

These materials will teach you the what and the how: what’s on the test and how to take the test. These materials will come from a test prep company (this is what test prep companies do!). You may decide to choose different materials from different companies, but I do recommend sticking with “sets” of materials whenever possible. For example, if you’re going to use the algebra study materials from one company, it’s best to use that company’s quant materials in general. Likewise on verbal.

2. Practice questions.

As you’re studying the material tested on the exam and how to handle the different types of GMAT questions, you’ll also need to test yourself on GMAT-format problems. The best practice questions are the officially released past test questions from GMAC (the makers of the GMAT). The latest three books are The Official Guide 13th Edition, the Verbal Review 2nd Edition and the Quantitative Review 2nd Edition. The most recent online release is GMAT Prep 2.0 (including 2 free practice tests and some additional paid resources) and there’s also GMAT Focus (for quant only).

3. Practice tests.

You’ll want a mix of practice tests: GMATPrep (from the real makers of the test) and some tests from a test prep company. The GMATPrep test is the closest to the real thing, but doesn’t offer explanations or analysis of your results. A test prep company’s CAT will give you explanations and analysis.

What’s next? Join us Friday for the second half of the series to learn how to learn!
This Blog post was imported into the forum automatically. We hope you found it helpful. Please use the Kudos button if you did, or please PM/DM me if you found it disruptive and I will take care of it. -BB
Manhattan GMAT Online Marketing Associate
Joined: 14 Nov 2013
Posts: 272
Own Kudos [?]: 51 [0]
Given Kudos: 0
Send PM
Re: Updates from Manhattan GMAT [#permalink]
Expert Reply
FROM Manhattan GMAT Blog: New Year’s Resolution: Get Your Score! (Part 2)

How do you study? More importantly, how do you know that the way in which you’re studying is effective—that is, that you’re learning what you need to learn to improve your GMAT score? Read on!

In the first part of this series, we discussed how to get started: setting up your timeframe, picking out your materials, and so on. (If you haven’t read it yet, please do so before you continue here!) In today’s installment, we’ll talk about how to study and make progress over the actual length of your study timeframe.

HOW Do I Learn?
This section addresses probably the single biggest mistake that people make when preparing for the GMAT.

At first, you’re going to concentrate more on what you need to learn / re-learn, but as you progress, you’re going to concentrate more on learning how to think. Yes, you need to know the formula for the area of a circle and how modifiers work and so on. You also need to know how to handle the different question types given on the GMAT.

But that’s only the start. Once you learn or re-learn a lot of that content, you will then need to move to the next level, which is what this test is really testing: how to think your way through any given problem, making the best possible decisions for each given situation. (Read that article I just linked.)

“Light bulb” Moments

You learn how to do this by analyzing the way these problems are put together by the test writers. You’ll actually learn to recognize what the test writers are trying to obscure, because you’ll have seen something like it before and you’ll have taken the time to think through it when the clock isn’t ticking.

Think about that the last time you were reading a new question and a “light bulb” went off in your head because you knew what to do. That was recognition! The more parts of new problems you can recognize, the better you’ll do on this test. Those of us who score in the 99th percentile don’t do so because we have some magic ability to figure everything out in three seconds. Rather, we’ve taught ourselves to recognize various bits of GMAT language, so that we have a huge advantage on most new questions.

Your goal is to learn to recognize as much as you can, so that you have as many “light bulb” moments as possible on test day.

Analyzing Problems

When doing GMAT-format problems, be aware that roughly 80% of your learning comes after you have finished doing the problem. Your goal here is not to do a million questions—your goal is to do a much more modest number of questions and really analyze them to death. Here’s how to review GMAT practice problems. You can find additional articles illustrating this process here, in the How To Study section.

I’ll repeat: you do not need to do every last OG problem out there. You do, however, need to learn something from each and every problem that you do—ideally multiple things. Otherwise, you are literally wasting your time!

The Plan
Okay, so you know your goal score, you know your strengths and weaknesses, and you’ve gathered your materials. You also know how to study: content / memorization, yes, but also a focus on how to think through problems. It’s time to develop your specific plan.

If you are taking a course, follow the syllabus. If you’re working with a tutor, figure out the plan with your tutor.

Otherwise, pick a time frame (generally two to three weeks) and decide what weaknesses you want to improve in that timeframe. In general, start with your biggest weaknesses in areas that are frequently tested on the GMAT. If you’re not sure which areas are most frequently tested, ask the experts on the forums. (I’m not listing them here because they can change over time.)

Get a calendar and block off one to two hours each day (okay, you can have one day off each week
). You don’t have to do your study all at once; you might do half an hour at lunch and another hour after work. Also, you’ll probably have some days on which you can study only 30 minutes or even 15. That’s fine—start off planning for 1-2 hours each day, but it’s okay if a few days “slip.” You may then have other days on which you study 3 or 4 hours; that’s fine as well, as long as you don’t study for more than about 2 hours in one sitting. (Why? Read this.)

In your journal, write down what your focus will be for each of the first six study days (one week). The first 5 sessions might, for example, consist of reading various chapters in various books and doing practice problems associated with those chapters. Estimate how much time you think it will take but be flexible; some study will go faster and some will take you longer than you expect.

Day 6 is a review day; you might do some sets of random problems, review what you did during the first 5 days, do a few problems from older areas that you haven’t studied recently, et cetera.

Individual Study Sessions

When you start a study session, pick an area of focus. Perhaps you’ll be working on linear and quadratic equations, or Find the Assumption questions, or Smart Numbers techniques for math. If you’re learning this material for the first time, start by working through whatever material you have that teaches you about that topic.

For example, if you’re using our materials to study Find the Assumption, you would read through the first Assumption Family chapter in your CR book. Do some exercises to test your understanding of the material you’re learning (in our book, these exercises are already built into the chapter).

When you feel you’ve got a grasp on the material, try a medium-level OG problem; then, move to a harder or easier one, depending on how you did. Review the official explanations as well as any alternate explanations that you find valuable—for example, you might look up the problem in our OG Archer program, or search for a discussion of the problem on the forums. Go over your work using the analysis techniques discussed in the next section. You may even want to return to the Find the Assumption questions you saw on your most recent practice tests and try them again.

At the end of each study session, jot down in your journal what you did that day, what you think went well, and what you think needs more work. (This knowledge will all come from your analysis of what you did that day.) If something didn’t go as well as you’d hoped, then feel free to adjust your calendar. At the end of the week, review your journal and set up your plan for the following week.

Study Strategy

During a particular study session, if you are reading lessons and then doing “skill drill” type practice problems (not GMAT format) in that same area, you should spend about 50% to 60% of your time learning and the rest drilling. If you are doing and then reviewing sets of GMAT-format practice problems, then you should spend at most 40% of your time doing a set of questions and at least 60% of your time reviewing those questions. (The 60%+ includes whatever you need to do in order to get better—re-read part of a chapter, figure out a more efficient way to do something, post a question on the forum, make up a couple of flashcards, etc.)

You’ll spend roughly the first 4 to 10 weeks focused more on the content (how does parallelism work, what’s an inference question, how do I solve simultaneous equations?). Perhaps 80% of your focus will be on content for the first few weeks, but you’ll gradually begin to add in the “how to think” aspect—in fact, the How To Analyze series of articles linked earlier is all about training ourselves how to think.

As you start finishing your test prep books / lessons (the ones that teach you the actual content), you’ll need to start focusing more heavily towards how to think about problems that test those content areas. You may start your session by doing a set of 10 mixed questions (not all the same type), after which you’ll analyze them all thoroughly and record your major takeaways, all of which can easily take 2 hours. While you’re doing that analysis, you’ll review any books or lessons necessary to get the most out of whatever problem you’re studying right now.

Quizzing and Testing Yourself
Periodically, quiz yourself. Mini-quizzes can be done a few times a week: a 5 or 10-minute flashcard quiz, for example, while you’re on the subway or waiting for that conference call to start. Regular quizzes should be done roughly once a week—a 5- to 10-question set of GMAT-format practice questions done under timed conditions, for example. (Don’t forget to analyze these thoroughly when you’re done!) As you progress through the test prep lessons (especially after you have been through all of the content material once), you may begin to do regular quizzes two or three times a week.

Repeat until you feel you’ve made good progress across multiple areas and are ready to test yourself on a CAT again. (This will typically take at least two to three weeks . Don’t take a CAT every week—that’s a waste of valuable study time!)

Do it all over again
Your overall process is going to be: take a CAT, analyze it, set up at least 2-3 weeks’ worth of work, then (when you feel ready) take another CAT and repeat the whole cycle.

When you take your second CAT, don’t worry about the overall score. Specifically check the areas on which you’d been concentrating for the previous several weeks. Most students’ scores stay the same or even go down on CAT 2 because there’s a pretty good chance you’ll mess up the timing in some way. (Plus, if you skipped essay and IR on the first test but did them on the second test, then don’t expect much improvement on the Q and V scores.)

For the areas that you did study, though—did they get better (though you may still be struggling on time or certain concepts)? Can you move on to other topics or question types, or are you still stuck in some areas? If you need advice about how to improve, log onto the forums again!

Then, review the overall test again (the same thing you did on the first test, way back in the first half of this article) and add the highlights of your analysis to your journal.

Next, if you haven’t yet done a first pass through your main study materials, continue on with the next thing on your list / syllabus. If you have been through all of your main study materials at least once (the stuff that teaches you what to do, from test prep companies), then your test results will tell you which areas to prioritize for review. If that’s the case, figure out what your new priorities are, set up your first 6-day plan, and repeat the whole process for several weeks until you feel ready for another test.

Take the test
Keep doing this until either your practice test scores are in your desired range or you hit a hard deadline and are forced to take the test even if your score isn’t quite where you want it yet. (And, in that case, accept that you may have to lower your goal.) If at all possible, study for the GMAT so far in advance of any deadlines that you don’t have to cut yourself off.

Good luck and happy studying!

Read New Year’s Resolution: Get Your Score! Part 1.
This Blog post was imported into the forum automatically. We hope you found it helpful. Please use the Kudos button if you did, or please PM/DM me if you found it disruptive and I will take care of it. -BB
Manhattan GMAT Online Marketing Associate
Joined: 14 Nov 2013
Posts: 272
Own Kudos [?]: 51 [0]
Given Kudos: 0
Send PM
Re: Updates from Manhattan GMAT [#permalink]
Expert Reply
FROM Manhattan GMAT Blog: Andrew Yang: “Smart People Should Build Things” Excerpt 4
Below is an excerpt from Andrew Yang‘s new book, Smart People Should Build Things: How to Restore Our Culture of Achievement, Build a Path for Entrepreneurs, and Create New Jobs in America, which comes out in February 2014. Andrew was named Managing Director of Manhattan GMAT in 2006, Chief Executive Officer in 2007, and President in 2010. He left Manhattan GMAT in 2010 to start Venture for America, where he now serves as Founder and CEO.


Professional Services as a Training Ground.  

As we’ve seen, one of the most frequently pursued paths for achievement-minded college seniors is to spend several years advancing professionally and getting trained and paid by an investment bank, consulting firm, or law firm. Then, the thought process goes, they can set out to do something else with some exposure and experience under their belts.  People are generally not making lifelong commitments to the field in their own minds. They’re “getting some skills” and making some connections before figuring out what they really want to do.

I subscribed to a version of this mind-set when I graduated from Brown. In my case, I went to law school thinking I’d practice for a few years (and pay down my law school debt) before lining up another opportunity.

It’s clear why this is such an attractive approach. There are some immensely constructive things about spending several years in professional services after graduating from college. Professional service firms are designed to train large groups of recruits annually, and they do so very successfully. After even just a year or two in a high-level bank or consulting firm, you emerge with a set of skills that can be applied in other contexts (financial modeling in Excel if you’re a financial analyst, PowerPoint and data organization and presentation if you’re a consultant, and editing and issue spotting if you’re a lawyer). This is very appealing to most any recent graduate who may not yet feel equipped with practical skills coming right out of college.

It seems like an incredible set of benefits. How can there be any downside either to the individual or to the economy that a significant proportion of our top graduates are being professionalized as bankers, consultants, or lawyers?

The nature of professional services dictates that you work on a deal or a client engagement that lasts a brief period and then ends. You’re usually staffed on a deal that will last for a finite period until the deal either comes through or falls apart. You begin a new transaction or client engagement every several months, perhaps longer if it’s a protracted consulting project. You’re used to relationships measured in weeks or months, or only hours or minutes in the trading context. Clients arrive and demand a flurry of activity until a transaction is complete, then disappear. Senior managers at your firm maintain relationships with clients, but you’re a level or two removed.  You often develop strong relationships with colleagues due to the long hours, extensive travel, and intense work environment. But you’re used to people coming and going very quickly as teams either shift and change or people leave the firm. For example, the attrition rate at one top consulting firm is 30 percent per year, which is one reason they’re always hiring.

The constant flow of different deals is presented as a selling point by many consulting firms and investment banks. They’ll say it’s “fast-paced,” things are “changing all the time,” and that you’ll work on one deal or project “and then move on.” Most operating companies, in contrast, typically rely upon long-term relationships to function well. They require a significant commitment in which the time frame is measured in years, not weeks or months. Turnover is detrimental to developing a good management team; building a business, and building up the value of one’s equity and relationships within an industry, takes time.

As a professional service provider who is changing clients or transactions every period, it’s hard to become emotionally invested in your work. It’s like trying to be concerned about taking care of a car you’re renting. Your clients are themselves big companies, and your interaction with them will often be limited to the occasional meeting with a senior executive or a manager. If you’re a consultant, you’re generally set up in a conference room from Monday through Thursday in a far-flung city; then you fly home on Thursday night. You’re there as a transaction cost because someone wants to get something done. One ex-consultant I interviewed noted, “It’s hard to get personally attached or invested when you know you’re only there for a number of months. I had assignments and deliverables that I knew would get changed after six months because we were a stopgap solution—I knew my work would disappear in a little while after the new system was put in.”

Your appetite for risk generally diminishes as you get older. This can become even more pronounced in a professional setting. You spend your working life in nice offices around well-compensated people.

You often have support staff from day one. The only people you interact with work at large public companies. Your expenses creep upward over time, and you get used to having nice things. Your interpersonal obligations mount, and the people you’re dating and family members expect you to earn lots of money. As you adapt to your role and circumstance, taking a risk professionally becomes more and more of an abstraction.

Once, while I was having drinks with a friend of mine after she started working at a top-tier consulting firm, she said, “Before I got here, I thought I could do anything. Now, I feel like you can’t do anything unless you have a budget of millions of dollars.”

In the minds of college seniors, and thanks to prodigious investment on the part of the firms themselves, professional services— financial services and management consulting—have become conflated with “business” when really they’re a narrow subset or category of businesses with distinctive features.

If you work in professional services you will be paid handsomely and have a brand-name firm on your résumé. You’ll gain skills, confidence, and exposure. But you may also become heavily socialized and specialized, more risk averse, and accustomed to operating in resource-rich environments with a narrow set of deliverables. You’ll be likely to adopt an arm’s-length relationship with your work. You won’t build anything; instead, you will compartmentalize and put the armor on each day as deals, clients, and colleagues come and go.

Professional services are being used as a de facto training ground for our top college graduates—with mixed results for everyone concerned. In particular, going into banking or consulting to learn how to start or run a business is not always ideal; the processes are very different, and give you a sense of companies trying to do different things. It’s like trying to learn how to become a chef by going to a company that runs analyses for large restaurant chains. Yes, you’ll get a better grasp of how chain restaurants work. But will you learn to cook?

There are, of course, any number of successful business builders and entrepreneurs who started out as professionals, as one would expect given that literally half our top graduates have pursued these paths for the past couple decades. David Gilboa worked at an investment bank before cofounding Warby Parker. John Delbridge worked in equity research before cofounding Mimeo. People have long careers that aren’t defined by their first few years.

And it’s easy to get excited about a potential hire if he has spent a couple of years at a top firm. There’s a good chance that this person is smart, motivated, capable of long hours and detail-oriented work, and is looking for a change. If applying to work at a startup, he probably expects a pay cut and has the right motivation.

But if I had a dollar for all the bankers, consultants, and lawyers I’ve met who told me that they were “really interested in entrepreneurship,” I’d be awfully rich. Meanwhile, they struggle to transition into different roles, and many of them have lost some of the qualities that would have enabled them to take on their original ambitions.

Their problem isn’t just theirs—it affects all of us. We’re breeding large battalions of indifferent professionals in a handful of cities when what we need is something very different. We need committed builders.

From SMART PEOPLE SHOULD BUILD THINGS by Andrew Yang© 2014 Andrew Yang. Reprinted courtesy of Harper Business, an imprint of HarperCollins Publishers.
This Blog post was imported into the forum automatically. We hope you found it helpful. Please use the Kudos button if you did, or please PM/DM me if you found it disruptive and I will take care of it. -BB
Manhattan GMAT Online Marketing Associate
Joined: 14 Nov 2013
Posts: 272
Own Kudos [?]: 51 [0]
Given Kudos: 0
Send PM
Re: Updates from Manhattan GMAT [#permalink]
Expert Reply
FROM Manhattan GMAT Blog: Advanced Critical Reasoning Lesson: RTFQ

So your Critical Reasoning (CR) score has moved a little, but not enough. Or each question is still taking you 3 minutes to answer. You’ve studied for months, read the Strategy Guides, taken every practice test, and completed every Critical Reasoning question in the big Official Guide and the Verbal Review supplement so many times you have them all memorized. What more can you do? Do more questions? You can probably imagine, more questions will usually mean more of the same issues, and simply reinforce bad habits…

Chances are, despite all your hard work, you’re still using your intuition and “gut feeling” to answer CR questions. Unfortunately, your gut feeling works some of the time, but not 100% of the time. Remember, the test is designed so that the average person picking what “looks right” will get only 50% of the questions correct.

So what to do? For now, stop doing more questions until you 1) learn the formal rules of logic behind how CR works, and 2) deeply analyze all the questions you’ve done for repeating patterns: question types, patterns of reasoning, logical flaws, right and wrong answer types, etc.

So that’s what the next few weeks will be about. Each week, I’ll post an article that goes absurdly in-depth about one aspect of the logic behind CR, along with exercises to apply those lessons. These are the same exercises I do with my tutoring students, who have found them very effective. I’m also interested in your feedback: what worked for you? What didn’t? Questions and concepts you’re still struggling with? I’m open to discussion and debate.

So let’s get started. I’ll start with the essentials and then really nerd out on formal logic, so keep reading to the end.

LESSON ONE: RTFQ

In our classes, we teach a four-step process to answering CR questions:

1) Identify the question (Know what the question is asking and what kind of question it is)

2) Deconstruct the argument (Analyze each piece of the passage for what role it plays)

3) Pause and state the goal (Predict what the correct answer should do)

4) Work from wrong to right (Use process of elimination to get to the right–or “least wrong”–answer.)

Today’s focus: Step one, which I call RTFQ, as in “Read the F___ Question” (F as in Full! Read the Full question. What were you thinking?)

The basics: The GMAT only asks a limited number of questions, with very rare variation. Each type of question implies HOW you should deconstruct the argument and WHAT the right answer will do. If you don’t identify the question properly, you won’t look for the right things, or you’ll waste time reading for things that aren’t there. So…Right now, can you name them all? No really.

Exercise 1: Before you scroll down, get out your notebook and write down as many types of questions as you can think of. Ready? Go.

.

.

.

.

.

How many did you come up with? 5? 6? Depending on how you break them down (what books you’ve read and who taught you), there are anywhere from 10-13 common types of questions. Here’s a list of the 11 most common that I use, grouped by category.

Structure based:

Identify the bolded part (role in the reasoning)

Identify the overall reasoning

Identify the conclusion

Mimic the reasoning (also known as parallel the reasoning)

Reasoning/assumption based:

Assumption

Strengthen

Weaken (and Flaw questions)

Evaluate

Fill in the Blank

Evidence or fact-based questions:

Inference (also known as “Draw a Conclusion” questions)

Resolve or Explain (a paradox or discrepancy)

I’ll explain more about the categories in future articles, but for now… Can you identify them when they show up? One of the most common mistakes you can make on the GMAT is simply misidentifying the question (e.g. mistaking strengthen for inference or strengthen for explain).

Exercise 2: Pick a dozen questions and name them! Take out your Official Guide for GMAT Review and get to work. Let’s pick numbers 50 through 61. If you need help, skim the passage itself to (All questions excerpted from The Official Guide for GMAT Review 13th Edition, by GMAC®)

50. Which of the following, if true, most strengthens the argument above?

51. The argument is most vulnerable to the objection that it fails to

52. Which of the following, if true, most strongly supports Summit’s explanation of its success in retaining employees?

53. Which of the following strategies would be most likely to minimize company X’s losses on the policies?

54. If the statements above are true, which of the following must be true?

55. Which of the following most logically completes the argument given below?

56. The conclusion above would be more reasonably drawn if which of the following were inserted into the argument as an additional premise?

57. Which of the following, if true, most helps to explain the surprising finding?

58. Which of the following, if true, most seriously weakens the conclusion above?

59. Which of the following most logically completes the passage?

60. If the facts stated in the passage above are true, a proper test of a country’s ability to be competitive is its ability to

61. Which of the following, if true, does most to explain the contrast described above?

And for good measure, identify number 66.

66. Which of the following conclusions about Country Z’s adversely affected export-dependent industries is best supported by the passage?

Write down what you think each one is.

Ready for your answers? Here’s how I identified them, with commentary.

50. Strengthen: Pretty straight up. The correct answer will strengthen the argument above.

51. Weaken: Or more specifically, identify the flaw in the reasoning. The words “it fails to” mean that the right answer, when considered, will damage the argument.

52. Strengthen: Don’t let the word “explain” fool you. The explanation is already in the argument; in fact the explanation may be the conclusion of the argument. Your job is to find an additional piece of evidence to strengthen that explanation.

53. Resolve/Explain: This one was tough. The question implies that there’s a problem (losses) to be solved (“minimize[d]“), which is what many resolve/explain questions do. Also, the argument itself describes a pretty clear contradiction: how does X keep its prices low, but also make enough income to pay for claims? The answer will resolve this. Feel free to argue with me in the comment section, though.

54. Inference (also known as “draw a conclusion”): Notice how “the statements above are true.” That mean you WON’T be looking for premises and conclusions, just putting facts together to find out what else must be true. More about this later in the section about “Deductive Reasoning.”

55. Fill in the Blank: note that the blank part starts with the word “because____” so you’ll be providing a premise that helps the conclusion. So, in a way, you can look at this as a strengthen question, too.

56. Assumption: Yes, assumption, though if you named this as a strengthen question, you’ll probably get it right. Technically, though, when the GMAT asks for an additional or unstated premise that makes the argument “more reasonably drawn” or that is “required,” it’s asking you for the assumption. But it’s interesting to note that assumption and strengthen questions both do the same thing: support the reasoning of an argument.

57. Resolve/explain: NOT strengthen. Imagine walking into your house to find your favorite chair is broken. Explaining WHY it’s broken is far different from Strengthening or fixing the chair with additional support.

58. Weaken: fair enough, easy to spot.

59. Fill in the blank: and with the word “since____” leading off the blank, it’s another strengthen.

60. Inference: Again, “if the statements above are true…” your reading for facts, not arguments.

61. Resolve/explain: again

aaaaand #66?

66. Inference: Yes. Inference. NOT STRENGTHEN! For more about how to differentiate between Inference and Strengthen questions, see our Critical Reasoning Strategy Guide, chapter 6.

So, how’d you do? If you were less than 100%, spend some time with the strategy guide, focusing on how to identify question types. Write down several examples of each question type and quiz yourself some more. You can use the Official Guide Problem Sets in the back of the CR Strategy Guide to see whether you were right or not. Keep working until you’re 100%.

NERDING OUT ON LOGIC

Critical reason is a test of LOGIC. So, with a big stack of logic books next to me, I’m going to discuss some of the formal rules behind the what GMAT writes questions. Ready?

The GMAT uses the word conclusion in two different ways. Most of the time, the GMAT is referring to an “inductive” conclusion, but occasionally, it’s asking about a “deductive” conclusion. Don’t confuse them!

So to explain: There are two kind of reasoning in the word: deductive reasoning and inductive reasoning.

Deductive reasoning is more concrete, more mathematical, more “true.”

Wikipedia’s definition of deductive reasoning: “Deductive reasoning links premises with conclusions. If all premises are true, the terms are clear, and the rules of deductive logic are followed, then the conclusion reached is necessarily true.”

In other words, if the premises are true, then the conclusion must be true. Does this sound like a common question type? (Hint: it starts with an “I____”)

Here are some examples of deductively valid arguments.

Premise: Sally is taller than Frank.

Premise: Frank is taller than William

Conclusion: Sally must be taller than William.

(Other deductively valid conclusions: Frank is shorter than Sally. William is not the same height as sally.)

Premise: All cats are persnickety

Premise: Mr. Whiskers is a cat.

Conclusion: Mr. Whiskers is persnickety.

(Other deductively valid conclusions: Some persnickety things are cats. At least one cat is named Mr. Whiskers.)

Inductive reasoning is a little softer, and much more common on the GMAT and in the real world. Science, economics, medicine, and our justice system are largely based on induction.

Wikipedia’s definition again: “Inductive reasoning is reasoning in which the premises seek to supply strong evidence for (not absolute proof of) the truth of the conclusion. While the conclusion of a deductive argument is supposed to be certain, the truth of an inductive argument is supposed to be probable, based upon the evidence given.”

In other words, if the premised are true, then the conclusion has a probability of being true, but also a probability of being false.

I’m usually sleepy after 11:00pm.

It’s past midnight.

I must be sleepy.

3 out of 4 dentists recommend chewing OctiDent after meals.

You should chew Octident after every meal.

After I cut bacon out of my diet, I lost 5 pounds.

If you want to lose weight, you should cut bacon out of your diet.

Inductively valid arguments have a very high probability of being true, with little chance of contradictory evidence (good scientific theories). Inductively invalid arguments have a high probability of being false (horoscopes). The dividing line between valid and invalid arguments can be shady and can depend on context. 90% certainty would be a great bet at a casino, but a lousy bet on airplane guidance systems.

We’ll get more into how to evaluate inductive reasoning vs. deductive reasoning in later articles, but for now, lets just learn to spot it.

Exercise: peruse the Official Guide questions 50-61 again. Decide whether the question and argument will be based on induction or deduction (Hint: if the argument can be helped or hurt, it’s probably induction. In the conclusion must be true, it’s deduction.)

50. Which of the following, if true, most strengthens the argument above?

51. The argument is most vulnerable to the objection that it fails to

52. Which of the following, if true, most strongly supports Summit’s explanation of its success in retaining employees?

53. Which of the following strategies would be most likely to minimize company X’s losses on the policies?

54. If the statements above are true, which of the following must be true?

55. Which of the following most logically completes the argument given below?

56. The conclusion above would be more reasonably drawn if which of the following were inserted into the argument as an additional premise?

57. Which of the following, if true, most helps to explain the surprising finding?

58. Which of the following, if true, most seriously weakens the conclusion above?

59. Which of the following most logically completes the passage?

60. If the facts stated in the passage above are true, a proper test of a country’s ability to be competitive is its ability to

61. Which of the following, if true, does most to explain the contrast described above?

Answer Key:

50. Induction

51. Induction

52. Induction

53. Induction

54. DEDUCTION

55. Induction

56. Induction

57. Induction (The explanation will be inductively valid.)

58. Induction

59. Induction

60. DEDUCTION

61. Induction

What do you think about question 66? Discuss and debate it in the comments below!

Take some time looking up deductive reasoning vs. inductive reasoning on the web. Wikipedia is a good place to start. Then, start analyzing other questions for the kind of reasoning tested on each. You may find that a lot of the questions you got wrong were one type or the other.

For an advanced drill, dig up all the Inference questions you can find. (I’ll give you a few: 66, 91, 103, and 104) Some of them are asking you for deductively valid conclusions, while others are asking for inductively valid conclusions. Can you determine which is which? Again, post your results in the comments section below.

Get to work, and for now just focus on those questions! See you in future articles.
This Blog post was imported into the forum automatically. We hope you found it helpful. Please use the Kudos button if you did, or please PM/DM me if you found it disruptive and I will take care of it. -BB
Manhattan GMAT Online Marketing Associate
Joined: 14 Nov 2013
Posts: 272
Own Kudos [?]: 51 [0]
Given Kudos: 0
Send PM
Re: Updates from Manhattan GMAT [#permalink]
Expert Reply
FROM Manhattan GMAT Blog: Want a 51 on Quant? Can you answer this problem?

Sequence problems aren’t incredibly common on the test, but if you’re doing well on the quant section, be prepared to see one. Now, you’ve got a choice: do you want to guess quickly and save time for other, easier topics? Do you want to learn some “test savvy” techniques that will help you with some sequence questions but possibly not all of them? Or do you want to learn how to do these every single time, no matter what?

That isn’t a trick question. Every good business person knows that there’s a point of diminishing returns: if you don’t actually need a 51, then you may study for a lower (but still good!) score and re-allocate your valuable time elsewhere.

Try this GMATPrep® problem from the free test. After, we’ll talk about how to do it in the “textbook” way and in the “back of the envelope” way.

* ”For every integer k from 1 to 10, inclusive, the kth term of a certain sequence is given by 
. If T is the sum of the first 10 terms in the sequence, then T is

“(A) greater than 2

“(B) between 1 and 2

“(C) between 1/2 and 1

“(D) between 1/4 and 1/2

“(E) less than 1/4”

First, let’s talk about how to do this thing in the “textbook math” way. If you don’t want to do this the textbook math way, feel free to skip to the second method below.

Textbook Method

If you’ve really studied sequences, then you may recognize the sequence as a particular kind called a Geometric Progression. If not, you would start to find the terms and see whether you can spot a pattern.

Plug in k = 1, 2, 3. What’s going on?



What’s going on here? Each time, the term gets multiplied by -1/2 in order to get to the next one. When you keep multiplying by the same number in order to get to the next term, then you have a geometric progression.

This next part gets into some serious math. Unless you really just love math, I wouldn’t bother learning this part for the GMAT, because there’s a very good chance you’ll never need to use it. But, if you want to, go for it!

When you have a geometric progression, you can calculate the sum in the following way:



Next, you’re going to multiply every term in the sum by the common ratio. What’s the common ratio? It’s the constant number that you keep multiplying each term by to get the next one. In this case, you’ve already figured this out: it’s – 1/2.

If you multiply this through all of the terms on both sides of the equation, you’ll get this:



Does anything look familiar? It’s basically the same list of numbers as in the first sum equation, except it’s missing the first number, 1/2. All of the others are identical!

Subtract this second equation from the first:



The right-hand side of the equation is always going to be just the first term of the original sum. The rest of the terms on the right-hand side of the two equations are identical, so when you subtract, they become zero and disappear.

Solve for s:



This value falls between 1/4 and 1/2, so the answer is (D).

Back of the Envelope Method

There is another way to tackle this one. At the same time, this problem is really tricky—so this solution is still not an “easy” solution. Your best choice might be just to guess and move on.

Before you start reading the text, take a First Glance at the whole thing. It’s a problem-solving problem. The answers are… weird. They’re not exact. What does that mean?

Read the problem, but keep that answer weirdness in mind. The first sentence has a crazy sequence. The question asks you to sum up the first 10 terms of this sequence. And the answers aren’t exact… so apparently you don’t need to find the exact sum.

Take a closer look at the form of the answers. Notice anything about them?

They don’t overlap! They cover adjacent ranges. If you can figure out that, for example, the sum is about 3/4, then you know the answer must be (C). In other words, you can actually estimate here—you don’t have to do an exact calculation.

That completely changes the way you can approach this problem! Here’s the sequence:



According to the problem, the 10 terms are from k = 1 to k = 10. Calculating all 10 of those and then adding them up is way too much work (another clue that there’s got to be a better way to do this one). So what is that better way?

Since you know you can estimate, try to find a pattern. Calculate the first two terms (we had to do this in the first solution, too).



What’s going on? The first answer is positive and the second one is negative. Why? Ah, because the first part of the calculation is -1 raised to a power. That will just keep switching back and forth between 1 and -1, depending on whether the power is odd or even. It won’t change the size of the final answer, but it will change the sign.

Okay, and what about that second part? it went from 1/2 to 1/4. What will happen next time? Try just that part of the calculation. If k = 3, then just that part will become
.

Interesting! So the denominator will keep increasing by a factor of 2: 2, 4, 8, 16 and so on.

Great, now you can write out the 10 numbers!


… ugh. The denominator’s getting pretty big. That means the fraction itself is getting pretty small. Do I need to keep writing these out?

What was the problem asking again?

Right, find the sum of these 10 numbers. Let’s see. The first number in the sequence is 1/2 and the second is -1/4, so the pair adds up to 1/4.



Right now, the answer would be right between D and E. Does the sum go up or down from here?

The third number will add 1/8, so it goes up:



But the fourth will subtract 1/16 (don’t forget that every other term is negative!), pulling it back down again:



Hmm. In the third step, it went up but not enough to get all the way to 1/2. Then, it went down again, but by an even smaller amount, so it didn’t get all the way back down to 1/4.

The fifth step would go up by an even smaller amount (1/32), and then it would go back down again by yet a smaller number (1/64). What can you conclude?

First, the sum is always growing a little bit, because each positive number is a bit bigger than the following negative number. The sum is never going to drop below 1/4, so cross off answer (E).

You keep adding smaller and smaller amounts, though, so if the first jump of 1/8 wasn’t enough to get you up to 1/2, then none of the later, smaller jumps will get you there either, especially because you also keep subtracting small amounts. You’re never going to cross over to 1/2, so the sum has to be between 1/4 and 1/2.

The correct answer is (D).

As I mentioned above, you may decide that you don’t want to do this problem at all. These aren’t that common—many people won’t see one like this on the test. Also, you don’t have to get everything right to get a top score. Just last week, I spoke with a student who outright guessed on 4 quant problems, and she still scored a 51 (the top score).

Key Takeaways for Advanced Sequence Problems

(1) Do you even want to learn how to do these? Don’t listen to your pride. Listen to your practical side. This might not be the best use of your time.

(2) All of these math problems do have a textbook solution method—but you’d have to learn a lot of math that you might never use if you try to learn all of the textbook methods. That’s not a problem if you’re great at math and have a great memory for this stuff. If not…

(3) … then think about alternate methods that can work just as well. Certain clues will indicate when you can estimate on a problem, rather than solving for the “real” number. You may already be familiar with some of these, for instance when you see the word “approximately” in the problem or answer choices that are spread pretty far apart. Now, you’ve got a new clue to add to your list: answers that offer a range of numbers and the different answer ranges don’t overlap.

* GMATPrep® questions courtesy of the Graduate Management Admissions Council. Usage of this question does not imply endorsement by GMAC.
This Blog post was imported into the forum automatically. We hope you found it helpful. Please use the Kudos button if you did, or please PM/DM me if you found it disruptive and I will take care of it. -BB
Manhattan GMAT Online Marketing Associate
Joined: 14 Nov 2013
Posts: 272
Own Kudos [?]: 51 [0]
Given Kudos: 0
Send PM
Re: Updates from Manhattan GMAT [#permalink]
Expert Reply
FROM Manhattan GMAT Blog: Andrew Yang: “Smart People Should Build Things” Excerpt 5
Below is an excerpt from Andrew Yang‘s new book, Smart People Should Build Things: How to Restore Our Culture of Achievement, Build a Path for Entrepreneurs, and Create New Jobs in America, which comes out in February 2014. Andrew was named Managing Director of Manhattan GMAT in 2006, Chief Executive Officer in 2007, and President in 2010. He left Manhattan GMAT in 2010 to start Venture for America, where he now serves as Founder and CEO.


Entrepreneurship Isn’t About Creativity.  

There is a common and persistent belief out there that entrepreneurship is about creativity, that it’s about having a great idea. But it’s not, really. Entrepreneurship isn’t about creativity. It’s about organization building—which, in turn, is about people.

I sometimes compare starting a business to having a child. You have a moment of profound inspiration, followed by months of thankless hard work and waking up in the middle of the night. People focus way too much on the inspiration, but, like conception, having a good idea isn’t much of an accomplishment. You need the action and follow-through, which involves the right people, know-how, money, resources, and years of hard work.

I learned this the hard way. Here’s a list of things you can reasonably do on the side as you’re working a full-time job to explore an idea for a great new business:

1. Research your idea (figure out the market, talk to prospective customers about what they would like, see who your competitors are, and so forth).

2. Undertake legal incorporation and trademark protection (the latter when necessary; most companies don’t need a trademark at first).

3. Claim a web URL and build a website or have it built; get company e-mail accounts.

4. Get a bank account and credit card (you’ll generally have to use personal credit at first).

5. Initiate a Facebook page, a blog, and a Twitter account if appropriate.

6. Develop branding (e.g., get a logo designed, print business cards).

7. Talk it up to your network; try to find interested parties as cofounders, staff, investors, and advisers.

8. Build financial projections and draft a business plan (if necessary).

9. Engage in personal financial planning (e.g., cut back on expenses, budget for startup costs, and so on.)

10. Create a mock prototype and presentation for potential investors or customers.

If all of this sounds like a lot of work to do before you’ve even really gotten started, you’re right. Getting this stuff done while holding down a job would be a significant commitment. You might not have time to hang out with friends and family and do the things people like to do when they’re not at work.  It is doable, though; I’ve seen it done or done it myself.

You’re just getting started. There’s a big jump in difficulty when it comes to the next things:

1. Raise money. In my experience, fledgling entrepreneurs focus way too much on the money—you can get most things done and figure out a lot without spending much. That said, most businesses require money to launch and get off the ground. For example, the average restaurant costs about $275,000 in construction and startup costs.  Finding initial funds is the primary barrier most entrepreneurs face. Many people don’t have three or six months’ worth of savings to free themselves up to do months of unpaid legwork.

2. Develop the product. Product development is a significant endeavor. Even if you’re hiring someone to build your product, managing them to specifications is a huge task in itself. You can expect vendors to take twice as long and cost twice as much as you’ve planned for. Think of the last home improvement project you paid a contractor for; most experiences are like that. Depending on the product, you may need to travel to find the right ingredients, partners, and suppliers. This phase might require raising additional money as well. In some cases, you might want to patent your product, which will involve a patent search and thousands of dollars in patent attorney fees.

3. Build a team. Most people don’t build a business alone, and finding quality partners or employees can be time-consuming and unpredictable. Your first employee is going to look to you for guidance, and her productivity is going to depend on your ability to guide and manage. And with partners, you’ll need to make sure you can work well with them, since they’re going to be with you from the ground up and for years afterward. 

4. Get customers. Going to trade shows or trying to get your first handful of paying customers is typically a major time investment. This can involve web marketing, producing content, and search engine optimization, all of which take significant energy and resources to generate a return. Despite the advent of social media, most things gain traction and spread at a deliberate pace. Even if someone likes your service, it’s not going to be a priority for him to go around telling friends about it or liking your service on Facebook.  Think about your own behavior; when’s the last time you went around telling everyone you know about a company you liked? Getting early sales is very hard work. You’ll likely depend on relationships to help you get the ball rolling.

You could give most people a fantastic business idea, and they would get excited about it but wouldn’t quit their job to take it on. And no one has an extra ten people in the back room waiting for a good idea either. How many great ideas have you had through the years that have remained just that thing you dreamed up with your friends that one time?

Starting a new business is generally going to be a multiyear commitment at the very least. One of my mentors, Manu Capoor of MMF Systems, once told me that it takes at least four or five years to see if a company is going to work. If you’re exceptional, you can tell where you’re going by year three. My experience has shown me that in almost every case, he was right.

If you just read business articles and blogs, you’ll get the sense that tons of companies enjoy immediate success, particularly in the Internet realm. But those are the anomalies. For most, overnight success is an extreme rarity. Generally, a company makes progress incrementally. Someone (or multiple people) likely suffered while figuring out how to make it work. Even for the rare product or software application that does become a rapid hit, it often took the programmers, product developers, or designers time to build up the necessary expertise. They might have worked on some earlier product that no one ever heard of, learned from it, and come back to build something great. This is a good description of Rovio, which was around for six years and underwent layoffs before the “instant” success of the Angry Birds video game franchise. In the case of the Five Guys restaurant chain, the founders spent fifteen years tweaking their original handful of restaurants in Virginia, finding the right bun bakery, the right number of times to shake the french fries before serving, how best to assemble a burger, and where to source their potatoes before expanding nationwide.

Most businesses require a complex network of relationships to function, and these relationships take time to build. In many instances you have to be around for a few years to receive consistent recognition. It takes time to develop connections with investors, suppliers, and vendors. And it takes time for staff and founders to gain effectiveness in their roles and become a strong team.*

*Experienced entrepreneurs have a number of advantages where pace is concerned. First, they know roughly how long it will take to get something done if they’ve done it before. Second, they can move faster, because many of the necessary relationships are already in place (e.g., they can call people they’ve worked with, use the same lawyer, accountant, and public relations firm, draw on earlier investors, and reach out to past customers). Third, they can proceed more decisively because of greater confidence in their judgment, both internally and externally. Last, they sometimes have lots of money. These are all reasons why some entrepreneurs seem so prolific.

From SMART PEOPLE SHOULD BUILD THINGS by Andrew Yang© 2014 Andrew Yang. Reprinted courtesy of Harper Business, an imprint of HarperCollins Publishers.
This Blog post was imported into the forum automatically. We hope you found it helpful. Please use the Kudos button if you did, or please PM/DM me if you found it disruptive and I will take care of it. -BB
Manhattan GMAT Online Marketing Associate
Joined: 14 Nov 2013
Posts: 272
Own Kudos [?]: 51 [3]
Given Kudos: 0
Send PM
Re: Updates from Manhattan GMAT [#permalink]
1
Kudos
2
Bookmarks
Expert Reply
FROM Manhattan GMAT Blog: ADVANCED CRITICAL REASONING, Part II: Deductive Logic


My last article discussed the difference between inductive and deductive arguments. Today’s article will focus mostly on the rules of deductive arguments. I promise to nerd out on inductive reasoning in later articles.

Here’s a quick quiz on the difference between inductive and deductive logic: https://www.thatquiz.org/tq/previewtest?F/Z/J/V/O3UL1355243858

To review: In a deductively “valid” argument, if all the premises are true, the conclusion must also be true, with 100% certainty. Luckily, on the GMAT, we should usually act as if the premises of an argument are true, especially when the question specifies, “the statements above are true.”

Deductive reasoning shows up most often on inference (aka “draw a conclusion”) questions and “mimic the reasoning” questions, but it often appears on other types of questions, and even on reading comprehension!

On inference questions, the correct answer will usually be deductively valid (or very very strong, inductively). An incorrect answer will be deductively invalid, with some significant probability that it could be false.

What follows are most of the formal rules of deductive reasoning (from a stack of logic textbooks I have on my shelf), with examples from the GMAT. For shorthand, I’ll label the arguments with a “P” for premise and a “C” for conclusion:

P) premise

P) premise

C) conclusion

Remember: these are not the same kind of conclusions (opinions) you’ll see on strengthen and weaken questions. Deductive conclusions are deductively “valid” facts that you can derive with 100% certainty from given premises.

EASY STUFF: Simplification/conjunction (“and” statements)

This is kind of a “duh” conclusion, but here goes: If two things are linked with an “and,” then you know each of them exist. Conversely, if two things exist, you can link them with an “and.”

Simplification:

P) A and B

C) Therefore, A

Conjunction:

P) A

P) B

C) Therefore, A and B

P) Bill is tall and was born in Texas.

P) Bill rides a motorcycle.

C) Therefore, Bill was born in Texas (simplification).

C) Therefore, at least one tall person named Bill was born in Texas and rides a motorcycle (conjunction).

CAUTION: Fallacies ahead!!

Don’t confuse “and” with “or.” (More about this later.) More importantly, don’t confuse “and” with causality, condition, or representativeness. Bill’s tallness probably has nothing to do with Texas, so keep an eye out for wrong answers that say, “Bill is tall because he was born in Texas” or “Most people from Texas ride motorcycles.”

MEDIUM STUFF: Disjunctive syllogism (“or” statements)

With “or” statements, if one thing is missing, the other must be true.

Valid conclusions:

P) A or B

P) not B (shorthand: ~B)

C) Therefore, A

P) We will go to the truck rally or to a Shakespeare play

P) We won’t go to the Shakespeare play.

C) Therefore, we will go to the truck rally.

CAUTION: Fallacies ahead!!

Unlike in the real world, “or” statements do not always imply mutual exclusivity, unless the argument explicitly says so. For example, in the above arguments, A and B might both be true; we might go to a play and go to the movies. Yes, really. A wrong answer might say “We went to a play, so we won’t go to the movies.” This error is called “affirming the disjunct.”

Invalid:

P) A or B

P) B

C) Not A

GMAT example:

To see this in action, check out your The Official Guide for GMAT Review 13th Edition, by GMAC®*, question 41. This argument opens with an implied “or” statement:

“Installing scrubbers in smokestacks and switching to cleaner-burning fuel are the two methods available to Northern Power…”

The author here incorrectly assumes that by using one method, Northern Power can’t use both methods at the same time. Question 51 does the same thing; discuss it in the comments below?

TOUGH STUFF: Fun with conditional statements

This is important! Keep a sharp eye out for statements that can be expressed conditionally and practice diagramming them. Look for key words such as “if,” “when,” “only,” and “require.”

I use the symbol “–>” to express an if/then relationship, and a “~” to express the word “not.” Use single letters or abbreviations to stand in for your elements.

If/then statements:

If you jump into that mud, you will get dirty: J –> D

If you don’t stop, I will faint: ~S –> F

I will scream if I hear that Bieber song again: B –>S

I will go only if you buy me dinner: Go –> Din

(Hint, replace the words “only if” with the arrow. See necessary/sufficient below.)

Extreme categorical statements (all, none, every, each, only, always, never):

I always go bowling on Tuesdays: T –> B

Every dog has ears: D –> E

Only teenagers listen to Bieber: B  –> T (notice that “only” is backwards from “every”)

No Librarians are Constructivists: L –> ~C

None of my friends eat sushi: F –> ~S

“or” statements:

I will order the cake or the pie: ~C –> P (and ~P –> C)

If you run across the word “unless,” it might help to replace it with “if not”:

I will show up to the barbecue unless its raining.

(“If not” raining, then BBQ): ~R –> B

Necessary/Sufficient statements (need, required, guarantee)

Remember this: Sufficient (guarantee, enough) goes on the left; Necessary (need, requirement) on the right

Sufficient –> Necessary

A good party needs beer: P –> B

A Katy Perry album guarantees a good time: KP –> GT

GMAT example:

Check out CR question 60 from the Official Guide for GMAT Review 13th Edition, by GMAC®. Brackets mine:

“Neither a rising standard of living [RSL] nor balanced trade [BT], by itself establishes a country’s ability to compete[C] in the international marketplace. Both are required simultaneously…”

Diagram this: C –> RSL & BT (both are necessary)

DON’T diagram this: RSL or BT –> C (each is sufficient)

Now, answering the question should be easy. Go for it.

VALID CONCLUSIONS FROM CONDITIONAL STATEMENTS

There are only a few valid deductions one can make from conditionals, and MANY invalid ones. Obviously, you won’t be tested on the Latin names, so worry more about the rules themselves and how they apply

Modus Tollens

Latin for “method that affirms by affirming,” this one more or less repeats the conditional statement as given:

P) A –> B

P) A

C) Therefore, B

If you think that’s too easy, check out Official Guide Question 60 again. It uses Modus Tollens!

P) A –> B & C

P) A

C) Therefore, B & C

Modus Ponens (the “contrapositive”)

EXTREMELY COMMON! Latin for “method that denies by denying,” this shows up all over the GMAT.

P) A –> B

P) ~B

C) Therefore, ~A

P) If you’re in Auckland, you’re in New Zealand

P) You’re not in New Zealand

C) Therefore, you’re not in Auckland

I (and many others) call this the contrapositive. To find the contrapositive, “flip and negate.” Just swap the elements and change negatives to positives:

X –> Y

~Y –> ~X

If you’re a libertarian, you’re not a communist: L –> ~C

Therefore: C –> ~L (If you’re a communist, you’re not a libertarian)

If you jump into that mud, you will get dirty: J –> D

~D –> ~J

If you don’t stop, I will faint: ~S –> F

~F –> S

Try diagramming the contrapositive for all the examples you’ve seen so far.

Advanced note: If a conditional contains an “and” or an “or,” change “and” to “or” and vice versa in the contrapositive. Remember to negate everything.

A –> B or C

(If I get a raise, I’ll go on vacation or buy a car.)

~B and ~C –> ~A

(I didn’t buy a car AND I didn’t go on vacation, so you know I didn’t get a raise.)

This works well with necessary/sufficient reasoning:

A good party needs beer and chips (remember, necessary elements go on the right):

P –> B and C

Therefore, ~B or ~C –> ~P

No beer? Not a good party. No chips? Not a good party.

GMAT example:

The Official Guide for GMAT Review 13th Edition, by GMAC®, question 103. Brackets mine:

“For a trade embargo [TE]…to succeed, a high degree of both international accord [IA] and ability to prevent goods [PG]…must be sustained.”

I diagram it like this: TE –> IA and PG

Then I do the contrapositive: ~IA or ~PG –> ~TE

If one of those elements is missing, you can’t have a trade embargo–much like our party without chips above. Work out the rest of the question for yourself.

Hypothetical Syllogism

Easy enough. You can chain if/then statements if they work out left to right:

P) A –> B

P) B –> C

C) Therefore, A –> C

GMAT Example:

This is a free question from the GMATPrep® software v.2.1*:

Increases in the level of high-density lipoprotein (HDL) in the human bloodstream lower bloodstream-cholesterol levels by increasing the body’s capacity to rid itself of excess cholesterol. Levels of HDL in the bloodstream of some individuals are significantly increased by a program of regular exercise and weight reduction.

Which of the following can be correctly inferred from the statements above?

(A) Individuals who are underweight do not run any risk of developing high levels of cholesterol in the bloodstream.

(B) Individuals who do not exercise regularly have a high risk of developing high levels of cholesterol in the bloodstream late in life.

(C) Exercise and weight reduction are the most effective methods of lowering bloodstream cholesterol levels in humans.

(D) A program of regular exercise and weight reduction lowers cholesterol levels in the bloodstream of some individuals.

(E) Only regular exercise is necessary to decrease cholesterol levels in the bloodstream of individuals of average weight.

You can chain the premises using conditionals as follows

Premise) incHDL –> lowBCL (B –> C)

Premise) someEX –> incHDL (A –> B)

Conclusion) Therefore, someEX –> lowBCL (A –> C)

Which is, more or less, the correct answer a nutshell. Work it out for yourself.

(Keep an eye out for words like “some” and “most” by the way)

CAUTION: Fallacies ahead!!

One common way the GMAT constructs wrong answers (and incorrect assumptions) is to mess up conditional logic in some way or another. Wrong answers will flip without negating, negate without flipping, confuse necessary with sufficient, mess up syllogisms, and make a series of either/or mistakes.

Flipping without negating (Affirming the consequent)

Invalid:

P) A –> B

P) B

C) Therefore, A

If you’re in Auckland, you’re in New Zealand. You’re in New Zealand. Therefore, you must be in Auckland.

(Nope, you might be in Auckland, but there are lots of other places you could be in New Zealand other than Auckland: Wellington, Nelson, Hobbittown, etc.)

This one is common in politics as well as on the GMAT:

No democrats are republicans (D –> ~R). You’re not a republican, so you must be a democrat (R –>~D).

(Nope, there are a lot of other political parties in the world…)

Negating without flipping (Denying the antecedent)

Invalid:

P) A –> B

P) ~A

C) ~B

If you’re in Auckland, you’re in New Zealand. You’re not in Auckland, so you can’t be in New Zealand.

No democrats are republicans (D –> ~R). You’re not a democrat, so you must be a republican (~D –> R).

Confusing necessary with sufficient or sufficient with necessary:

A good party needs chips and beer. We have chips and beer, so it’s going to be a good party.

(Nope, there may be other necessary requirements to a good party, such as music, a place to have the party, actual other people…)

Chopping off your leg is a guarantee that you’ll lose 30 pounds. Bill lost 30 pounds, so he must have chopped off his leg.

(There are other sufficient ways to lose 30 pounds.)

GMAT examples:

The Official Guide for GMAT Review 13th Edition, by GMAC®, question number 103: Remember this?

“For a trade embargo [TE]…to succeed, a high degree of both international accord [IA] and ability to prevent goods [PG]…must be sustained.”

One of the wrong answers says:

(B) As long as international opinion is unanimously against Patria, a trade embargo is likely to succeed.

Which is a whole lot like saying: “I have chips, so it’s going to be a good party!”

Even on other kinds of questions, the GMAT will confuse necessary/sufficient in wrong answers:

From the GMATPrep® 2.1 CAT exam practice test*, on one assumption question, the argument states:

“The interview is an essential part of a successful hiring program”

(Interview is necessary: SHP –> Int.)

Whereas one of the wrong answers states:

“A hiring program will be successful if it includes interviews.”

(An interview is sufficient: Int. –> SHP)

Syllogism Fallacies:

Be careful how you link syllogisms. Make sure they chain up correctly.

Invalid:

P) A –> B

P) A –> C

C) B –> C

Many wrong answers do this in weird ways. See Official Guide question 103:

“(E) For a blockade of Patria’s ports to be successful, international opinion must be unanimous.”

Either/Or fallacies (Affirming the disjunct)

We’ve covered this already, but to sum up: When you see “or” statements on the GMAT, pay attention to the precise phrasing.

Are the two things mutually exclusive (Cats and Dogs)? If so, do those two categories account for everything in the universe? Or are there possibilities of being other things? If so, you might want to diagram it as follows:

C –> ~D

D –> ~C

So it would be invalid to say: “That’s not a dog, so it must be a cat.” (~D –> C). You never know, it might be a wombat or the Empire State Building.

Seems obvious, but people do it all the time: (You’re not a democrat, so you must be a republican.)

On the other hand, is it a simple “or” statement that leaves the possibility of both things being true? (See disjunctive syllogism way earlier). “A or B” should be diagramed as:

~A –> B

~B –> A

Premise: My light doesn’t work. Either the power is out or the bulb is blown.

(~P or ~B)

Valid conclusion: The power is working, so the bulb must be blown. (P, so ~B)

Invalid conclusion: The bulb is blown, so the power must be working. (~B, so P) (Both things could be true)

Again, check out Official Guide questions 41 and 45!

IN CONCLUSION

Overwhelmed? Don’t be. The most important rule to remember is this:

Premise) A –> B

Conclusion) ~B –> ~A

Wrong) B –> A, ~A –> B

Otherwise, I just wanted to expand your mind a little, make you aware of the ways in which the GMAT construct right and wrong answers, and to give you some tools to deeply analyze tough Critical Reasoning questions.

From now on when you struggle with a CR question, try to figure out which logical fallacy the test writers used to construct each wrong answer (during the review process, NOT on the test itself).

For example: take the HDL problem we discussed earlier. You now know how the correct answer was written. What about the incorrect ones? Can you spot the fallacies at work? Please discuss in the comments section.

Increases in the level of high-density lipoprotein (HDL) in the human bloodstream lower bloodstream-cholesterol levels by increasing the body’s capacity to rid itself of excess cholesterol. Levels of HDL in the bloodstream of some individuals are significantly increased by a program of regular exercise and weight reduction.

Which of the following can be correctly inferred from the statements above?

(A) Individuals who are underweight do not run any risk of developing high levels of cholesterol in the bloodstream.

(B) Individuals who do not exercise regularly have a high risk of developing high levels of cholesterol in the bloodstream late in life.

(C) Exercise and weight reduction are the most effective methods of lowering bloodstream cholesterol levels in humans.

(D) A program of regular exercise and weight reduction lowers cholesterol levels in the bloodstream of some individuals.

(E) Only regular exercise is necessary to decrease cholesterol levels in the bloodstream of individuals of average weight.

Have fun! There’s a cool quiz here: https://www.think-logically.co.uk/lt.htm

More to come. If there are specific issues or questions you want me to cover, let me know.

* GMATPrep® questions courtesy of the Graduate Management Admissions Council. Usage of this question does not imply endorsement by GMAC.

* The text excerpted above from The Official Guide for GMAT Review 13th Edition is copyright GMAC (the Graduate Management Admissions Council). The short excerpts are quoted under fair-use statutes for scholarly or journalistic work; use of these excerpts does not imply endorsement of this article by GMAC.

Primary source: Critical Thinking, by Jamie Carlin Watson and Robert Arp. Continuum International Publishing.
This Blog post was imported into the forum automatically. We hope you found it helpful. Please use the Kudos button if you did, or please PM/DM me if you found it disruptive and I will take care of it. -BB
GMAT Club Bot
Re: Updates from Manhattan GMAT [#permalink]
 1   2   3   4   5   6   7   8   9   10   11  ...  21   

Powered by phpBB © phpBB Group | Emoji artwork provided by EmojiOne